¿Qué fuerza causa la FEM inducida de un bucle? Y, ¿cuál es la diferencia entre un EMF de bucle y un EMF de movimiento?

Si un bucle de alambre se coloca perpendicular a un campo magnético cambiante, hay una FEM inducida de tal manera que la corriente inducida fluye en una dirección para crear un campo magnético opuesto como este diagrama simple:

ingrese la descripción de la imagen aquí

Mientras que una varilla (o alambre) que se mueve en un campo magnético uniforme experimenta una fuerza magnética que empuja las cargas positivas hacia arriba para crear un EMF como este diagrama:

ingrese la descripción de la imagen aquí

Preguntas:

1) La barra tiene una fuerza magnética debido a su movimiento en B que causa la EMF inducida, para un campo magnético cambiante perpendicular a un bucle (como el primer diagrama), ¿hay una fuerza magnética que también crea la EMF inducida? Si no, ¿cuáles son las fuerzas que causan la FEM inducida?

2) Para la FEM de movimiento de una varilla, al aumentar la velocidad, ¿cómo aumenta la FEM inducida? La expresion ϵ = v B L es simple pero imaginarlo es confuso. ¿Al tener más cargas? ¿Para un campo E más fuerte?

3) Si la varilla se mueve en un campo magnético no uniforme, ¿se considera un campo B variable? ¿Es válida la fórmula para la FEM de movimiento? Mi enfoque es calcular B en ciertos puntos de X y usa la fórmula. ¿Es esto válido?

4) Esto se relaciona con la primera pregunta, para este diagrama:

ingrese la descripción de la imagen aquí

Un lazo moviéndose en un uniforme. B , tiene un EMF inducido = 0, debido a que los EMF de cada lado del cable se cancelan así:

ingrese la descripción de la imagen aquí

También están en serie, para mí es más fácil imaginar esos cables como fuentes de batería, todos con el mismo potencial opuesto entre sí. ¿Por qué no es esto similar al primer diagrama de una variación B ? El bucle rectangular que se mueve en un campo magnético tiene cero EMF, mientras que un bucle en un campo magnético cambiante tiene un EMF distinto de cero, ¿por qué los lados no se cancelan?

@Lelouch hice una pregunta sobre el mismo tema ( physics.stackexchange.com/questions/525498/… ). haría

Respuestas (1)

  1. El caso de un conductor móvil y un conductor estacionario es fundamentalmente diferente. Cuando el conductor está estacionario, un campo magnético cambiante produce un campo eléctrico en todo el espacio, cuyo giro a lo largo de cualquier bucle que encierra el campo magnético variable es distinto de cero, dado por × mi = B t . Usando la ley de Stokes, encontramos fácilmente que la fem es la tasa de cambio del flujo a través del bucle. NOTA: es el campo eléctrico producido en el espacio el que causa la fem (en este caso), no cualquier fuerza.
  1. La fem a través de un conductor de extremo abierto (supongamos que los extremos son A y B) SOLO se debe al campo electrostático conservativo producido por las cargas separadas, debido a la fuerza magnética, dada por la integral del campo electrostático sobre la longitud del conductor. para A a B: mi = mi s d yo . A medida que el conductor se acelera, debido a la mayor fuerza magnética, se deposita una mayor carga en los extremos, lo que resulta en una mayor magnitud del campo electrostático ( mi s ) a través del conductor, que a su vez aumenta su integral sobre la longitud (¡que es la fem, por supuesto!).

  2. Nuevamente, para un conductor abierto, tomamos el campo electrostático, que es q v B / q = v B . Sustituyendo en la fórmula e integrando, mi = v B L . Entonces, la fórmula sigue siendo válida.

  3. Estás confundiendo de nuevo el campo eléctrico no conservativo mi con el campo electrostático conservativo ( mi s ). Cuando el circuito está cerrado, la fem: mi = mi d yo , dónde mi es el campo total (debido tanto a la electricidad como a la electrostática en todo el circuito). La parte electrostática es obviamente cero en todo el bucle (porque es conservativa), pero el campo eléctrico viene dado por la ecuación de Maxwell: × mi = B t . para constante B , × mi = 0 , que a partir de la ley de Stokes sugiere que su integral alrededor del bucle también es cero, lo que produce una fem cero. Para un campo magnético variable, la integral del campo electrostático se desvanece de todos modos, pero × mi no es cero, lo que hace que la fem no sea cero en la integral. No introduzca un campo electrostático en fem en un circuito cerrado. Te confundirás.

Formatee sus expresiones usando MathJax. Este es un sitio habilitado para MathJax; para obtener ayuda, verifique esto: Tutorial básico de MathJax y referencia rápida .
Ok lo haré seguro.
@Lelouch ¡Gracias! Ahora entendí la diferencia entre un campo E electrostático (EsEs) como el cable (o bucle) que se mueve en un BB que no cambia y un campo eléctrico no conservativo (EE) de un BB variable. He editado la publicación con algunas preguntas más, ¿podría responderlas?